- Joined
- Apr 2025
- Subscription
- Free
Hey thanks for your response! I definitely better understand necessary assumptions now, and I think your point about an assumption for the argument vs. for the conclusion is where my issue was. A necessary assumption is needed for the support structure of the argument to work - it's required for the premises to support the conclusion.
Another way to think of it (and where my issue was) is with the negation test. As Kevin Lin put it in his response to my comment in lesson 1 (thanks Kevin!), "the negation does not need to show that the conclusion cannot be true. It just needs to show that the given premises, without more, are not enough to guarantee that the conclusion is true." So negating the correct answer choice does leave open the possibility that the chair didn't endorse for a different reason. However, when it comes to the argument structure, negating the correct answer choice makes it so that the premises that we have no longer support the conclusion.
After a LOT of thought, I think I came up with a good explanation for why AC A is the correct answer for Question 15:
The final paragraph seems to have a sub-conclusion and a main conclusion: The sub-conclusion is the line that says "Hence creating a link to a document is not the same as making or distributing a copy of that document" (line 42). This lends support to the main conclusion in the last sentence: "Changing copyright law to benefit owners of intellectual property is thus ill-advised" (line 52).
At first glance, it looks like AC A really just captures the idea presented in the sub-conclusion: that creating a link to a document does not take away any control of distribution from the author, and therefore creating such a link is not considered copyright infringement.
But wait, that's not what A says. It doesn't say that, therefore, "creating such a link is not considered copyright infringement." It says that "creating such a link should not be considered copyright infringement."
It looks like, then, this second part of AC A actually does capture the meaning (albeit somewhat implicitly) of the main conclusion presented in the passage's very last sentence. The passage states that "changing copyright law to benefit owners of intellectual property is thus ill-advised." In saying "creating such a link should not be considered copyright infringement," AC A is saying that the copyright laws should not be altered to extend to the prohibition of the creation of links to documents. It can be viewed as saying that "creating such a link should not be considered copyright infringement by new/changed laws."
AC A essentially uses the descriptive sub-conclusion (starting with "since") to support its prescriptive main conclusion (starting with "creating"), which is exactly what the last paragraph did.
This makes a lot of sense and saves me a lot of confusion. I much better understand these types of questions now. Thanks so much for your response, I appreciate it!
Sorry, meant to write "just because there is a conditional statement, it doesn't meant that there are not other routes..."
When applying the negation test, does it mean that negating the correct answer choice would render the conclusion impossible, or just that the premises would no longer support the conclusion? It makes more sense to me that it would mean that the negation of the answer choice would mean that the conclusion can absolutely no longer be drawn, but there's a few questions in the upcoming lessons where it seems like that isn't the case. Instead, it seems like if the correct answer choice weren't true, that the premises just no longer directly support the conclusion. However, negating the answer choice doesn't seem to preclude the possibility of other assumptions being made that could allow the conclusion to be drawn.
Someone please help lol. After writing out "endorsement → [certain conditions]," JY says "[James] did something to make this conditional endorsement to go away. And what's the thing he could have done? The only thing he could have done?" And then says that James must have failed the necessary condition. But isn't it true that failing the necessary condition is not the only way to come to the conclusion of /endorsement? The contrapositive gives us /[certain conditions] → /endorsement. So fulfilling the /[certain conditions] sufficient condition is sufficient to come to the conclusion of /endorsement, but I thought we learned that just because there isn't a conditional statement, it doesn't mean that there are not other routes that can be taken to get to necessary condition. I.e., just because A → B doesn't mean that X → B can't also be true. So as it relates to this question, I don't see how answer choice E is necessary. I see how it's sufficient to make the argument valid and brings you to the conclusion of /endorsement, but aren't there other avenues that also could have been taken to get to the conclusion of /endorsement? For instance, can't it be true that maybe the proposal that the chair had seen actually did include all of the recommendations (meaning E is false), but the chair still decided not to endorse for another reason? Like maybe she was just in a bad mood that day? The argument doesn't seem to exclude this possibility. It only looks like it would exclude this possibility if the conditional had been "if and only if" rather than just "if."
I would really appreciate some help!! I'm really struggling to see how this is a necessary assumption and not just a sufficient assumption. Thank you!!
So is this technically a PSAr question or a Principle question? It came up in my drill for PSA questions, but the lack of the word "justify" makes it seem like it's a Principle question, which I haven't even gotten to yet in the syllabus.
The second rule can be contraposed, but it just would never be able to bring you to the conclusion that something can not be justified. If we contraposed the second rule, we would get:
/J → /(Comp TOB and RGBEL)
This then becomes:
/J → /Comp TOB or /RGBEL, meaning:
If an action is not justified, then either it was not a computer typically used in the operation of a business, or there did not exist reasonable grounds for believing that such a computer contained data usable as evidence in a legal proceeding against the computer's owner.
From this rule, we can never come to the conclusion that an action is not justified. If the sufficient condition is fulfilled (/J), then we can logically come to the conclusion (/Comp TOB or /RGBEL). But If the necessary condition is fulfilled, that tells us nothing. If we fulfill the necessary condition, we cannot say that the sufficient condition must be true. We cannot reverse the direction of the arrow.
If we fail the necessary condition, then we know that the sufficient condition is failed. But that conclusion would bring us to "J," meaning that the action was justified. So again, this second rule can never bring us to the conclusion of /J.
Because the conclusions in all of the answer choices are either "the action was justified" or "the action was not justified," the only conclusions we're concerned about when writing out our logic are J and /J. This is probably why the instructor did not bother contraposing the second rule. Not because it cannot be done, but because it would not bring us to any conclusions that we need.
The rules are written out in lawgic from two separate premises. The instructor isn't really just taking the necessary condition from one rule and making it the sufficient condition of another. Rather, he's writing the rules out separately as direct translations from the stimulus.
The first premise tells us that "one is justified in accessing information in computer files without securing authorization from the computer's owner only if the computer is typically used in the operation of a business." "Only if" is a group 2 indicator word, meaning that it's giving us a necessary condition. This premise means that "if someone is justified in accessing info without authorization, then the computer must be typically used in the operation of a business." This gives us:
J → Comp TOB
The next premise tells us that "if there exist reasonable grounds for believing that such a computer contains data usable as evidence in a legal proceeding against the computer's owner, then accessing the data without authorization is justified." The referential word "such" is referring to "computers typically used in the operation of a business," which was stated previously in the first premise. This is what "such" is referring to. Essentially, this second premise is saying "if there exist reasonable grounds for believing that a computer typically used in the operation of a business contains data usable as evidence in a legal proceeding against the computer's owner, then accessing the data without authorization is justified." With our second premise now written out in full, we can see that there's two parts of the sufficient condition needed to be fulfilled in order to trigger the conclusion. There needs to be reasonable grounds for believing the computer contains data usable as evidence, and that computer needs to be one that is typically used in the operation of a business. If both of these conditions are met, then accessing the data without authorization is justified. This gives us:
RGBEL and Comp TOB → J
#feedback I think there's a typo in the written explanation for answer choice E: "the stimulus already foreclosed the hypothesis that Travaillier's current customers' preferences have changed." Should that instead be "have not changed"?
My thoughts exactly! Like in the previous question, just because we know that doubling phosphorus is a cause of plankton growth, we can't imply that there are no other causes of plankton growth (and therefore no other causes of few fish surviving). Just because the cause is not fulfilled does not mean the effect will not be fulfilled.
When I read it too quickly, I thought D was a restatement of what was said in the stimulus too, but it's not. The stimulus basically says that, for several decades, every time the number of transistors has doubled, the the computing speed has doubled. Then from about 1995-2005, every time the computing speed has doubled, the production cost has doubled. When A happens, B happens. And when B happens, C happens. Therefore, when A happens, C happens.
Take this with a grain of salt, but after a lot of thought, I think I was able to articulate to myself the reasoning for E being the correct answer (with the video's help of course):
If Wally’s is a large nursery, then what do we know to be true? We know that Wally’s probably (because of the “most” statements regarding large nurseries), sells primarily to commercial growers. And we know that Wally’s probably sells plants that are guaranteed to be disease-free. But how can this be? Maybe Wally’s is not part of the “most” nurseries that guarantee this. But that’s not probably what happened. (It is possible, because it’s not ruled out by the lawgic. But is that strongly supported by the stimulus? No. That’s actually less likely to be true because according to the stimulus, and given that Wally’s is a large nursery according to E, Wally’s most likely does have that guarantee.) Because most large nurseries have that guarantee, then Wally’s probably did have that guarantee (because according to E, we’re saying that Wally’s is a large nursery). So Wally’s probably had that guarantee. How can this be true? Well, Wally’s probably had that guarantee, but the guarantee probably failed. The raspberries were not actually as they were guaranteed to be.
Love this explanation lol. Only issue I see is that by kicking up coffeehouses and restaurants to the domain, we’re still left being susceptible to answer choice C (C and D both could be true if we do this). Aside from the conditional statements you’ve got in there, the stimulus also gives us WD ‑m→ FA. Using this premise, if we kicked up coffeehouses and restaurants to the domain, C’s claim that “most coffeehouses that are well designed feature artwork” would follow logically. I think we have to acknowledge that coffee houses and restaurants aren’t part of the domain, but are instead subsumed by the “public places” domain. That way, we can eliminate C by recognizing that it gives us an “all before most” or even a “some before most” conditional, from which we can’t draw the conclusion that C wants us to.
(bang)
In the explanation for choice C, it's mentioned that our chain tells us that "fish and /birds → gerbils." Therefore, we can infer that some stores that sell gerbils also sell fish but not birds (because an "all" statement in one direction implies a "some" statement in the other direction, or really both directions, because that's how "some" works).
I vaguely remember learning this in a foundations lesson, but now I'm wondering, what if there are no "fish and /birds" stores? Like, what if all the stores that sell fish but not birds closed down? We could still say that "if a store sells fish but not birds, it does sell gerbils," because that is given to us in the stimulus, and the validity isn't affected by anything. In a world where all of the stores that sell fish but not birds closed down, "fish and /birds → gerbils" could still be valid, but there just wouldn't be any of those particular stores to fulfill that rule.
But now, in that world, can we still say that "some stores that sell gerbils also sell fish but not birds"? Probably not, because we could have stores that sell gerbils, but all the stores that sell fish but not birds closed down. So none of the stores that sell gerbils also sell fish but not birds.
I understand the logic behind inferring a "some" statement from an "all" statement, but can't get over this one possibility. The only answer I can come up with is if the very presence of something as a sufficient condition in a conditional relationship implies the existence of that thing (i.e. "fish and /birds → gerbils" indicates that there are existing stores that sell fish but not birds.) Or I'm wondering if maybe this is only the case for "all" statements, but not "if, then" statements? Because with "if, then" statements, the very basis of the relationship is that "if" something is true, "then" another thing is true, which kind of opens the door for that sufficient condition to maybe not be true in some scenarios (right?). Like if we reworded our original premise to "if a store sells fish but not birds, then it sells gerbils." Well, what if a store doesn't sell fish but not birds. What if none of the stores sell fish and not birds? The rule could still exist in this world, but we're just waiting for a store to open up that actually fulfills it.
Or maybe I'm just way off in some other way. If anyone knows what I'm missing here, I'd really appreciate your help!
The last sentence of the stimulus states that the current habitat is not large enough to support any more than the 70-100 that it currently has.
It is currently not large enough to support any more. Therefore, in order to support more, it has to be larger.
If it has to be larger to support more than 70-100, then it definitely must be larger to support 250+ (given that 250 is more than 70-100).
Therefore, if there were 250+ panthers, the habitat must be larger:
250+ → larger habitat
The stimulus states "their population must reach at least 250 if it is to be self-sustaining."
In other words, "the population has to be 250+ in order to be self-sustaining," or "if the population is to be self-sustaining, then it must be 250+."
Now we can see that the sufficient condition is "self-sustaining" and the necessary condition is "250+"
Written in lawgic, we get "SS → 250+"
This is not the same as "250+ → SS." This would be confusing the sufficient condition for the necessary condition. This is simply not what the stimulus states.
The stimulus is telling us that, no matter what, if the population is self-sustaining, there MUST be 250+ panthers (SS → 250+), because that number is necessary for the population to be self-sustaining. However, that doesn't mean that, no matter what, if there are 250+ panthers, it will be self-sustaining. Based on what we have in the stimulus, there is the possibility that other things are needed for the population to be self-sustaining. Maybe there is not a big enough food source, or most of the panthers are actually infertile. If that were the case, even if there were 250+ of them, the population still couldn't be self-sustaining.
In other words, a population of 250+ is needed for it to be self-sustaining, but it does not guarantee that it will be self-sustaining.
Makes sense. Appreciate it!!
I feel kind of silly not just recognizing this as a "rule/exception to the rule" conditional type argument, but I did get the right answer and then in the blind review used lawgic to check the validity.
Using the indicator word "unless" to negate the sufficient, I got "if an antibiotic doesn't eliminate the species, the species becomes more resistant within a few years"
/eliminate → resistance
and with its contrapositive (just for funsies):
/resistance → eliminate
Then using the indicator word "no" in the second sentence, I got "if an antibiotic is currently on the market, it cannot eliminate species X"
current → /eliminate
contrapositive:
eliminate → current
Now if we connect some of these, we get:
current → /eliminate → resistance
Which is exactly what B says (current → resistance).
I feel like I'm not consciously doing this whole thing when I'm under the time constraint, so I'm not sure if I'm just lucking out or if I just know deep down in my heart of hearts what to do lol
I think I'm the same way. Reading the question first has helped me pay better attention to what I'm reading and understand what I'm looking for. Thanks for your comment!
Makes sense. Thanks!
The video explanation does do a good job at explaining why this answer choice is wrong - this answer choice is asserting that those people who eat chocolate essentially misreported their experience. But the written explanation confused me lol. I think that the written explanation is saying that the author doesn't challenge/argue against the people who eat chocolate. Instead, it argues against the widely held belief/common wisdom.
The explanation for choice A states that "The argument never mentions individuals who claim they experience acne outbreaks during high-chocolate periods." But the sentence in the stimulus right before the conclusion says, "many people who are susceptible to acne report that, in their own experience, eating large amounts of chocolate is invariably followed by an outbreak of that skin condition." How is this not mentioning individuals who claim they experience acne breakouts during high-chocolate periods"?
Sorry if this has been answered before, but for logical reasoning questions, should we read the question first before the stimulus so we know what to look for? Or just read the stimulus first?
Wooo! I'm glad I could help!!